www.vorkurse.de
Ein Projekt von vorhilfe.de
Die Online-Kurse der Vorhilfe

E-Learning leicht gemacht.
Hallo Gast!einloggen | registrieren ]
Startseite · Mitglieder · Teams · Forum · Wissen · Kurse · Impressum
Forenbaum
^ Forenbaum
Status Mathe-Vorkurse
  Status Organisatorisches
  Status Schule
    Status Wiederholung Algebra
    Status Einführung Analysis
    Status Einführung Analytisc
    Status VK 21: Mathematik 6.
    Status VK 37: Kurvendiskussionen
    Status VK Abivorbereitungen
  Status Universität
    Status Lerngruppe LinAlg
    Status VK 13 Analysis I FH
    Status Algebra 2006
    Status VK 22: Algebra 2007
    Status GruMiHH 06
    Status VK 58: Algebra 1
    Status VK 59: Lineare Algebra
    Status VK 60: Analysis
    Status Wahrscheinlichkeitst

Gezeigt werden alle Foren bis zur Tiefe 2

Navigation
 Startseite...
 Neuerdings beta neu
 Forum...
 vorwissen...
 vorkurse...
 Werkzeuge...
 Nachhilfevermittlung beta...
 Online-Spiele beta
 Suchen
 Verein...
 Impressum
Das Projekt
Server und Internetanbindung werden durch Spenden finanziert.
Organisiert wird das Projekt von unserem Koordinatorenteam.
Hunderte Mitglieder helfen ehrenamtlich in unseren moderierten Foren.
Anbieter der Seite ist der gemeinnützige Verein "Vorhilfe.de e.V.".
Partnerseiten
Weitere Fächer:

Open Source FunktionenplotterFunkyPlot: Kostenloser und quelloffener Funktionenplotter für Linux und andere Betriebssysteme
Forum "Folgen und Reihen" - Konvergenzradius
Konvergenzradius < Folgen und Reihen < eindimensional < reell < Analysis < Hochschule < Mathe < Vorhilfe
Ansicht: [ geschachtelt ] | ^ Forum "Folgen und Reihen"  | ^^ Alle Foren  | ^ Forenbaum  | Materialien

Konvergenzradius: Frage (beantwortet)
Status: (Frage) beantwortet Status 
Datum: 21:57 Do 16.10.2008
Autor: HansPhysikus

Aufgabe
Bereche den Konvergenzradius von

[mm] \summe_{n=2}^{\infty}\frac{(-1)^{n+1}(2n-3)!!}{2^n n!}z^n [/mm]

Hallo

Konvergenzradius, Definition:

p = [mm] \frac{1}{\stackrel{limsup}{n\rightarrow\infty}\wurzel[n]{|a_n|}} [/mm]


Aus der Definition des Konvergenzradius habe ich mir hergeleitet:

H := [mm] \stackrel{limsup}{n\rightarrow\infty}\wurzel[n]{|a_n|} [/mm]

ln(H) = [mm] \stackrel{limsup}{n\rightarrow\infty}\frac{1}{n}ln(|a_n|) [/mm] := D

Also   H = [mm] e^D [/mm]

p = [mm] \frac{1}{\stackrel{limsup}{n\rightarrow\infty}\wurzel[n]{|a_n|}} [/mm] = [mm] \frac{1}{H} [/mm] = [mm] e^{-D} [/mm]

Betrachte nun:

[mm] \stackrel{limsup}{n\rightarrow\infty}\frac{1}{n}ln\left(\left|\frac{(2n-3)!!}{2^n n!}\right|\right) [/mm]

diesen ausruck habe ich jetzt schon seitenweise umgeformt, komme aber auf keinen ausdruck, welchen ich abschätzen könnte.

Als Tipp haben wir vom Prof noch bekommen:
[mm] ln(n!)\approx [/mm] n ln(n)
[mm] ln(n!!)\approx \frac{n}{2} [/mm] ln(n)

(diese tipps habe ich beim umfrmen natürlich auch benutzt)

Evtl. sieht ja einer von Euch direkt, was man hier machen kann...

LG,
HP

        
Bezug
Konvergenzradius: Antwort
Status: (Antwort) fertig Status 
Datum: 01:22 Fr 17.10.2008
Autor: schachuzipus

Hallo HP,

> Bereche den Konvergenzradius von
>  
> [mm]\summe_{n=2}^{\infty}\frac{(-1)^{n+1}(2n-3)!!}{2^n n!}z^n[/mm]
>  
> Hallo
>  
> Konvergenzradius, Definition:
>  
> [mm] $p=\frac{1}{\limsup\limits_{n\rightarrow\infty}\wurzel[n]{|a_n|}}$ [/mm]
>  
>
> Aus der Definition des Konvergenzradius habe ich mir
> hergeleitet:
>  
> [mm] $H:=\limsup\limits_{n\rightarrow\infty}\wurzel[n]{|a_n|}$ [/mm]
>  
> [mm] $\ln(H)=\limsup\limits_{n\rightarrow\infty}\frac{1}{n}\ln(|a_n|):=D$ [/mm]
>  
> Also   [mm] $H=e^D$ [/mm]
>  
> [mm] $p=\frac{1}{\limsup\limits_{n\rightarrow\infty}\wurzel[n]{|a_n|}}=\frac{1}{H}=e^{-D}$ [/mm]
>  
> Betrachte nun:
>  
> [mm] $\limsup\limits_{n\rightarrow\infty}\frac{1}{n}\ln\left(\left|\frac{(2n-3)!!}{2^n n!}\right|\right)$ [/mm]
>  
> diesen ausruck habe ich jetzt schon seitenweise umgeformt,
> komme aber auf keinen ausdruck, welchen ich abschätzen
> könnte.
>  
> Als Tipp haben wir vom Prof noch bekommen:
>  [mm] $\ln(n!)\approx n\ln(n)$ [/mm]
>  [mm] $\ln(n!!)\approx \frac{n}{2}\ln(2)$ [/mm]
>  
> (diese tipps habe ich beim umfrmen natürlich auch benutzt)
>  
> Evtl. sieht ja einer von Euch direkt, was man hier machen
> kann...

Benutze die bekannten Logarithmusgesetze und die hinweise deines Profs:

[mm] $\frac{1}{n}\ln\left|\frac{(2n-3)!!}{2^n n!}\right|=\frac{1}{n}\cdot{}\left(\ln|(2n-3)!!|-\ln\left|2^n n!\right|\right)$ [/mm]

[mm] $=\frac{1}{n}\cdot{}\left(\ln|(2n-3)!!|-(n\ln(2)+\ln|n!|)\right)$ [/mm]

[mm] $\underbrace{\approx}_{\text{Hinweis}}\frac{1}{n}\cdot{}\left(\frac{2n-3}{2}\ln(2)-n\ln(2)-n\ln(n)\right)$ [/mm]

[mm] $=\frac{2n-3}{2n}\ln(2)-\ln(2)-\ln(n)=\frac{2n\left(1-\frac{3}{2n}\right)}{2n}\ln(2)-\ln(2)-\ln(n)$ [/mm]

[mm] $=\left(1-\frac{3}{2n}\right)\ln(2)-\ln(2)-\ln(n) [/mm] \ [mm] \longrightarrow (1-0)\ln(2)-\ln(2)-\infty=-\infty$ [/mm] für [mm] $n\to\infty$ [/mm]


> LG,
>  HP


Gruß

schachuzipus

Bezug
Ansicht: [ geschachtelt ] | ^ Forum "Folgen und Reihen"  | ^^ Alle Foren  | ^ Forenbaum  | Materialien


^ Seitenanfang ^
www.vorkurse.de
[ Startseite | Mitglieder | Teams | Forum | Wissen | Kurse | Impressum ]